Skip to content

Commit

Permalink
修改错误
Browse files Browse the repository at this point in the history
修改错误
  • Loading branch information
syvshc committed Jan 2, 2020
2 parents 5c514fc + d3c2766 commit 0822922
Show file tree
Hide file tree
Showing 7 changed files with 92 additions and 88 deletions.
4 changes: 2 additions & 2 deletions Appendix_A.tex
Original file line number Diff line number Diff line change
Expand Up @@ -35,7 +35,7 @@

我们希望证明一个更强的命题, 尽管实分析课程中已经介绍过相关概念, (但是我猜可能考完试就忘记了), 所以在这里也再次重复一遍:
\begin{Definition}
对一般的准序集$ (P, \lesssim) $, 若$ \forall x_1, x_2\in P, \exists x\in P : (x_1\lesssim x)\land (x_2\lesssim x) $, 则称$ P $\textbf{上定向}的; 若$ \forall x_1, x_2\in P, \exists x\in P : (x_1\gtrsim x)\land (x_2\gtrsim x) $, 则称$ P $\textbf{下定向}的. 若$ P $上定向或者下定向, 则称$ P $是一个\textbf{定向集}.
对一般的准序集$ (P, \lesssim) $, 若$ \forall x_1, x_2\in P, \exists x\in P\,((x_1\lesssim x)\land (x_2\lesssim x)) $, 则称$ P $\textbf{上定向}的; 若$ \forall x_1, x_2\in P, \exists x\in P\,((x_1\gtrsim x)\land (x_2\gtrsim x)) $, 则称$ P $\textbf{下定向}的. 若$ P $上定向或者下定向, 则称$ P $是一个\textbf{定向集}.

$ \alpha $是一个定向集时, 一个$ \alpha $-组称为一个\textbf{}. 当$ (x_i)_{i\uparrow\alpha} $$ (P,\lesssim) $上的一个网时, 形式地定义\textbf{上极限}
\[
Expand Down Expand Up @@ -227,7 +227,7 @@

\textbf{定义\,\ref{def:Hilbert空间}\,下的注记}\ \ 内积空间的完备化是Hilbert空间.
\begin{Proof}
设内积空间$ H $上的内积$ \lrangle{\cdot,\cdot} $诱导范数$ \norm{\cdot} $, 范数$ \norm{\cdot} $诱导度量$ d(\cdot,\cdot) $, 对度量空间$ (H,d) $作完备化$ (\hat{H},d) $, 那么任取$ H $中一Cauchy列$ (x_n)_{n\geqslant 1} $, 存在$ \hat{x}\in\hat{H} $使得$ d(x_n,\hat{x})to 0 $成立.
设内积空间$ H $上的内积$ \lrangle{\cdot,\cdot} $诱导范数$ \norm{\cdot} $, 范数$ \norm{\cdot} $诱导度量$ d(\cdot,\cdot) $, 对度量空间$ (H,d) $作完备化$ (\hat{H},d) $, 那么任取$ H $中一Cauchy列$ (x_n)_{n\geqslant 1} $, 存在$ \hat{x}\in\hat{H} $使得$ d(x_n,\hat{x})\to 0 $成立.

定义$ \hat{H} $上的范数$ \norm{x-y}:=d(x,y) $, 这一定义在$ H $上的限制即为$ H $中的范数, 由度量的连续性可知$ \norm{x_n}\to\norm{x} $. 再由范数可以定义一个内积
\[
Expand Down
10 changes: 5 additions & 5 deletions Appendix_B.tex
Original file line number Diff line number Diff line change
Expand Up @@ -791,13 +791,13 @@ \section{第4章习题}
\]
对上述的 $ n_{0} $, 令 $ F=\{ (\seq[n_{0}]{x}, 0, \dots, 0, \dots) : x_{i}\in\C, 1\leqslant i\leqslant n_{0} \}\subset\ell_{2} $. 则 $ F $ 有限维, 再令
\[
B = \left\{ x= (\seq[n_{0}]{x}, 0,\dots, 0\dots) : \abs{x_{i}}<\frac{1}{i} \right\}\subset A,
B = \left\{ x= (\seq[n_{0}]{x}, 0,\dots, 0\dots) : \abs{x_{i}}\leqslant\frac{1}{i} \right\}\subset A,
\]
$ B $ 中元素的前 $ n_{0} $ 项构成 $ \tilde{B} $, 即
\[
\tilde{B}=\left\{ x= (\seq[n_{0}]{x}) : \abs{x_{i}}<\frac{1}{i} \right\}
\tilde{B}=\left\{ x= (\seq[n_{0}]{x}) : \abs{x_{i}}\leqslant\frac{1}{i} \right\}
\]
显然 $ B $$ \tilde{B} $ 同构. 且 $ B $$ F $ 中闭, 又 $ \abs{x_{1}}^{2}+\cdots+\abs{x_{n_{0}}}^{2}\leqslant1+1=2 $, 故 $ B $$ F $ 中有界闭, 从而紧, 所以有 $ \tilde{B} $$ \C^{n_{0}} $ 中紧, 于是 $ \tilde{B} $ 预紧, 即存在 $ \C^{n_{0}} $ 中有限个点 $ y^{(1)}, y^{(2)}, \dots, y^{(k)} $, 其中
显然 $ B $$ \tilde{B} $ 同构. 且 $ B $$ F $ 中闭, 又 $ \abs{x_{1}}^{2}+\cdots+\abs{x_{n_{0}}}^{2}\leqslant1+1=2 $, 故 $ B $$ F $ 中有界闭, 从而紧, 所以有 $ \tilde{B} $$ \C^{n_{0}} $ 中紧, 于是 $ \tilde{B} $ 预紧, 即存在 $ \tilde{B} $ 中有限个点 $ y^{(1)}, y^{(2)}, \dots, y^{(k)} $, 其中
\[
y^{(i)}=(y^{(i)}_{1}, \dots, y^{(i)}_{n_{0}} ), \quad i = 1, 2, \dots, k
\]
Expand Down Expand Up @@ -851,14 +851,14 @@ \section{第4章习题}
\[
\begin{aligned}
\frac{1}{2^{n}}\sum_{(\varepsilon_{k})\in D_{n}}\norm{\sum_{k=1}^{n}\varepsilon_{k}x_{k}}^{2} & \leqslant \frac{M^{2}}{2^{n}}\sum_{(\varepsilon_{k})\in D_{k}}\abs{\sum_{i=1}^{n}\varepsilon_{i}x_{i}} \\
\frac{1}{2^{n}}\sum_{(\varepsilon_{k})\in D_{n}}\norm{\sum_{k=1}^{n}\varepsilon_{k}x_{k}}^{2} & \leqslant \frac{M^{2}}{2^{n}}\sum_{(\varepsilon_{k})\in D_{k}}\abs{\sum_{k=1}^{n}\varepsilon_{k}x_{k}}^{2} \\
& = M^{2}\left(\sum_{k=1}^{n}\abs{x_{k}}^{2}\right)\leqslant \frac{M^{2}}{m^{2}}\left(\sum_{k=1}^{n}\norm{x_{k}}^{2}\right)
\end{aligned}
\]
同理有
\[
\begin{aligned}
\frac{1}{2^{n}}\sum_{\left(\varepsilon_{k}\right)\in D_{n}}\norm{\sum_{k=1}^{n}\varepsilon_{k}x_{k}}^{2} & \geqslant \frac{m^{2}}{2^{n}}\sum_{\left(\varepsilon_{k}\right)\in D_{k}}\abs{\sum_{i=1}^{n}\varepsilon_{i}x_{i}} \\
\frac{1}{2^{n}}\sum_{\left(\varepsilon_{k}\right)\in D_{n}}\norm{\sum_{k=1}^{n}\varepsilon_{k}x_{k}}^{2} & \geqslant \frac{m^{2}}{2^{n}}\sum_{\left(\varepsilon_{k}\right)\in D_{k}}\abs{\sum_{k=1}^{n}\varepsilon_{k}x_{k}}^{2} \\
& = m^{2}\left(\sum_{k=1}^{n}\abs{x_{k}}^{2}\right)\geqslant \frac{m^{2}}{M^{2}}\left(\sum_{k=1}^{n}\norm{x_{k}}^{2}\right)
\end{aligned}
\]
Expand Down
59 changes: 30 additions & 29 deletions Appendix_B2.tex
Original file line number Diff line number Diff line change
Expand Up @@ -152,7 +152,7 @@ \section{第6章习题}
\[
\forall e\in B_E\,(\norm{e+\ker u}\leqslant\norm{e}<1)
\]
可知$ q(B_E)\subset B_{E/\ker u} $. 而$ \forall e+\ker u\in B_{E/\ker u} $, 有$ \norm{u+\ker u}<1 $, 则存在$ a\in\ker u $使得$ \norm{e+a}<1 $. 且$ q(e+a)=e+\ker u $, 从而$ e+\ker u\subset q(B_E) $. 由$ e+\ker u $的任意性可知$ B_{E/\ker u}\subset q(B_E) $. 从而$ B_{E/\ker u}=q(B_E) $.
可知$ q(B_E)\subset B_{E/\ker u} $. 而$ \forall e+\ker u\in B_{E/\ker u} $, 有$ \norm{e+\ker u}<1 $, 则存在$ a\in\ker u $使得$ \norm{e+a}<1 $. 且$ q(e+a)=e+\ker u $, 从而$ e+\ker u\subset q(B_E) $. 由$ e+\ker u $的任意性可知$ B_{E/\ker u}\subset q(B_E) $. 从而$ B_{E/\ker u}=q(B_E) $.

那么
\[
Expand Down Expand Up @@ -224,7 +224,7 @@ \section{第6章习题}
$ x_n=t^n $, 那么$ \norm{x_n}=1 $$ \norm{u(x_n)}=n\to\infty $, 从而$ u $不连续. 这说明当$ X $不完备时闭图像定理不成立.\qed
\end{Proof}

\textbf{习题 6.19}\ [习题课]\ \ $ (X, \CA, \mu) $$ \sigma $-有限的测度空间.
\textbf{习题 6.19}\ [习题课]\ \ $ (X, \CA, \mu) $$ \sigma $--有限的测度空间.
\begin{enumerate}[(1)]
\item 假设当 $ 1\leqslant p<q\leqslant\infty $ 时, 有 $ L_{q}(X, \CA, \mu)\subset L_{p}(X, \CA, \mu) $. 证明存在常数 $ C\geqslant0 $, 使得对任意的 $ f\in L_{q}(X, \CA, \mu) $, 有 $ \norm{f}_{p}\leqslant c\norm{f}_{q} $.
\item 导出下列命题的等价性
Expand All @@ -235,17 +235,17 @@ \section{第6章习题}
\end{enumerate}
\end{enumerate}
\begin{Proof}
(1) 因为 $ X $$ \sigma $-有限的, 故存在一列递增的可列集 $ (A_{n})_{n\geqslant1} $$ X=\bigcup_{n\geqslant1}A_{n} $, 且 $ \forall n\geqslant1:\mu(A_{n})<\infty $, 考虑线性映射:
(1) 因为 $ X $$ \sigma $-有限的, 故存在一列递增的可测集 $ (A_{n})_{n\geqslant1} $$ X=\bigcup_{n\geqslant1}A_{n} $, 且 $ \forall n\geqslant1:\mu(A_{n})<\infty $, 考虑线性映射:
\[
I_{n}: L_{q}(X)\to L_{p}(X), \quad f\mapsto f\cdot 1_{A_{n}}.
\]
则由习题 3.11 有
\[
\norm{I_{n}(f)}_{p}=\left( \int_{A_{n}}\abs{f}^{p}\diff\mu \right)^{1/p}\leqslant \mu(A_{n})^{1/p-1/q}\left( \int_{A_{n}\abs{f}^{q}}\diff\mu \right)^{1/q}=\mu(A_{n})^{1/p-1/q}\norm{f}_{q},
\norm{I_{n}(f)}_{p}=\left( \int_{A_{n}}\abs{f}^{p}\diff\mu \right)^{1/p}\leqslant \mu(A_{n})^{1/p-1/q}\left( \int_{A_{n}}\abs{f}^{q}\diff\mu \right)^{1/q}=\mu(A_{n})^{1/p-1/q}\norm{f}_{q},
\]
$ \norm{I_{n}}\leqslant\mu(A_{n})^{1/p-1/q} $, 则 $ \lim\limits_{n\to\infty}I_{n}=\id $, 由推论~\ref{cor:逐点收敛}~知 $ \id $ 连续, 且 $ \sup\limits_{n\leqslant1}\norm{I_{n}}<\infty $, 且 $ \norm{\id}\leqslant\liminf\limits_{n\to\infty}\norm{I_{n}} $. 记 $ c=\liminf\limits_{n\to\infty}\norm{f_{n}} $, 则 $ \norm{f}_{p}\leqslant c\norm{f}_{q} $.

(2) a $ \Rightarrow $ b. 用反证法, 设 $ \mu(X)=\infty $, 则 $ \forall n\geqslant1 $, 存在 $ \mu $-可列集 $ B_{n} $ 使得 $ \mu(B_{n})\geqslant n $, 取 $ f = 1_{B_{n}} $, 则
(2) a $ \Rightarrow $ b. 用反证法, 设 $ \mu(X)=\infty $, 则 $ \forall n\geqslant1 $, 存在 $ \mu $-可测集 $ B_{n} $ 使得 $ \mu(B_{n})\geqslant n $, 取 $ f = 1_{B_{n}} $, 则
\[
\norm{f}_{p}=\left( \int_{B_{n}}1\diff\mu \right)^{1/p}=\mu(B_{n})^{1/p}=n^{1/p}.
\]
Expand Down Expand Up @@ -276,7 +276,7 @@ \section{第8章习题}
\[
\tilde{f} = x_{1}\tilde{f}(e_{1})+x_{2}\tilde{f}(e_{2})=x_{1}+x_{2}\tilde{f}(e_{2}),
\]
$ \tilde{f} $ 由其在 $ e_{2} $ 上的取值唯一确定, 记 $ c=\tilde{f}(e_{2}) $, 由 $ \tnorm{\tilde{f}} $$ \abs{c}=\tabs{\tilde{f}(e_{2})}\leqslant\norm{e_{2}}=1 $. 反之, 对任意的 $ c $ 满足 $ \abs{c}<1 $, 形如 $ \tilde{f}(x)=x_{1}+x_{2}c $ 的泛函是 $ f $ 的保范延拓.
$ \tilde{f} $ 由其在 $ e_{2} $ 上的取值唯一确定, 记 $ c=\tilde{f}(e_{2}) $, 由 $ \tnorm{\tilde{f}}=1 $$ \abs{c}=\tabs{\tilde{f}(e_{2})}\leqslant\norm{e_{2}}=1 $. 反之, 对任意的 $ c $ 满足 $ \abs{c}<1 $, 形如 $ \tilde{f}(x)=x_{1}+x_{2}c $ 的泛函是 $ f $ 的保范延拓.

(2) 当 $ 1<p<\infty $ 时, 与 (1) 同理, 只需确定 $ c $ 使 $ \tnorm{\tilde{f}}=1 $, 则
\[
Expand Down Expand Up @@ -338,9 +338,9 @@ \section{第8章习题}
\]
先证$ \tilde{f} $well-defined. 若$ \sum\limits_{k=1}^n\alpha_ka_k=\sum\limits_{l=1}^n\beta_lb_l $, 其中$ a_k, b_l\in A $, 则由
\[
\abs{\sum_{k=1}^n\alpha_kf(a_k)-\sum_{l=1}^n\beta_lb_l}\leqslant\lambda\norm{\sum_{k=1}^n\alpha_ka_k-\sum_{l=1}^n\beta_lb_l}=0
\abs{\sum_{k=1}^n\alpha_kf(a_k)-\sum_{l=1}^n\beta_lf(b_l)}\leqslant\lambda\norm{\sum_{k=1}^n\alpha_ka_k-\sum_{l=1}^n\beta_lb_l}=0
\]
可知$ \tilde{f} $well-defined且$ \tilde{f} $连续, $ \tnorm{\tilde{f}}\leqslant\lambda $. 由Hahn-Banach定理可知$ \tilde{f} $可以延拓为$ E $上的连续线性泛函$ \hat{f} $, 且它满足$ \hat{f}\rvert_A $$ \tnorm{\hat{f}}\leqslant\lambda $.\qed
可知$ \tilde{f} $well-defined且$ \tilde{f} $连续, $ \tnorm{\tilde{f}}\leqslant\lambda $. 由Hahn-Banach定理可知$ \tilde{f} $可以延拓为$ E $上的连续线性泛函$ \hat{f} $, 且它满足$ \hat{f}\rvert_A=f $$ \tnorm{\hat{f}}\leqslant\lambda $.\qed
\end{Proof}

\textbf{习题8.4}\ [习题课]\ \ $ E $是Hausdorff拓扑向量空间, $ A $$ E $中包含原点的开凸集且$ x_0\in E\sm A $.
Expand Down Expand Up @@ -382,20 +382,21 @@ \section{第8章习题}
E=H+\K x_0=H+\K\frac{x-h}{\alpha}=H+\K\frac{-h}{\alpha}+\K\frac{x}{\alpha}=H+\K x.
\]

(2) 若$ H $稠密, 则命题得证. 若$ H $不稠密, 往证$ H $是闭集. 用反证法, 设$ (x_n)_{n\geqslant 1}]\subset H $使得$ x_n\to x_0\notin H $, 则$ E=H+\K x_0 $. 任取$ y\in E\sm H $, 那么$ y=h+\alpha x_0 $, 其中$ h\in H $, $ \alpha\in\K $, 则
\[
\begin{aligned}
d(y,H)=\inf_{z\in H}\norm{y-z}=\inf_{z\in H}\norm{h+\alpha x_0-z}&=\inf_{z\in H}\norm{\alpha x_0-(z-h)}\\&=\inf_{z\in H}\norm{\alpha x_0-z}=\abs{\alpha}\cdot d(x_0,H)=0
\end{aligned}
\]
最后一个等号因
\[
\forall\varepsilon>0\,\exists z\in H\,\exists n_0\in\N\,((d(x_0,z)<d(x_0,H)+\varepsilon)\land(n\geqslant n_0\Rightarrow d(x_n,x_0)<\varepsilon))
\]
这与$ y\notin H $矛盾, 从而$ x_0\in H $, 故$ H $是闭的.
(另证: 设$ H $是超平面, 则存在$ x_0\in E\sm H $使得$ E=H+\K x_0 $. 若$ x_0\in\bar{H} $, 则$ E=H+\K x_0\subset\bar{H} $, 则$ E=\bar{H} $, 故$ H $稠密. 若$ x_0\notin \bar{H} $, 则存在$ f\in\Star{E} $使得$ f\rvert_H=0 $$ f(x_0)\ne 0 $. 则$ H\subset\ker f $. 又因为$ \forall x\in E $$ x=h+\alpha x_0 $, 则$ f(x)=0\Rightarrow\alpha f(x_0)=0\Rightarrow \alpha=0 $, 故$ x=h\in H $. 这说明$ \ker f=H $, 则$ H $是闭的.)
(2) 设$ H $是超平面, 则存在$ x_0\in E\sm H $使得$ E=H+\K x_0 $. 若$ x_0\in\bar{H} $, 则$ E=H+\K x_0\subset\bar{H} $, 则$ E=\bar{H} $, 故$ H $稠密. 若$ x_0\notin \bar{H} $, 则存在$ f\in\Star{E} $使得$ f\rvert_H=0 $$ f(x_0)\ne 0 $. 则$ H\subset\ker f $. 又因为$ \forall x\in E $$ x=h+\alpha x_0 $, 则$ f(x)=0\Rightarrow\alpha f(x_0)=0\Rightarrow \alpha=0 $, 故$ x=h\in H $. 这说明$ \ker f=H $, 则$ H $是闭的.

% 若$ H $稠密, 则命题得证. 若$ H $不稠密, 往证$ H $是闭集. 用反证法, 设$ (x_n)_{n\geqslant 1}\subset H $使得$ x_n\to x_0\notin H $, 则$ E=H+\K x_0 $. 任取$ y\in E\sm H $, 那么$ y=h+\alpha x_0 $, 其中$ h\in H $, $ \alpha\in\K $, 则
% \[
% \begin{aligned}
% d(y,H)=\inf_{z\in H}\norm{y-z}=\inf_{z\in H}\norm{h+\alpha x_0-z}&=\inf_{z\in H}\norm{\alpha x_0-(z-h)}\\&=\inf_{z\in H}\norm{\alpha x_0-z}=\abs{\alpha}\cdot d(x_0,H)=0
% \end{aligned}
% \]
% 最后一个等号因
% \[
% \forall\varepsilon>0\,\exists z\in H\,\exists n_0\in\N\,((d(x_0,z)<d(x_0,H)+\varepsilon)\land(n\geqslant n_0\Rightarrow d(x_n,x_0)<\varepsilon))
% \]
% 这与$ y\notin H $矛盾, 从而$ x_0\in H $, 故$ H $是闭的.

(3a) \textsl{充分性}. 由题设可知存在$ x_0\in E\sm H $使得$ f(x_0)=0 $$ H=\ker f $, 则任取$ e\in E $都有
(3a) \textsl{充分性}. 由题设可知存在$ x_0\in E\sm H $使得$ f(x_0)\ne0 $$ H=\ker f $, 则任取$ e\in E $都有
\[
e=e-\frac{f(e)}{f(x_0)}x_0+\frac{f(e)}{f(x_0)}x_0\Longrightarrow E=H+\K x_0,
\]
Expand Down Expand Up @@ -427,7 +428,7 @@ \section{第8章习题}
\end{aligned}
\]
\begin{enumerate}[(1)]
\item 证明: $ A_0 $$ B $都是$ \ell_1 $的线性子空间, 且$ A_0+B=\ell_1 $中稠密;
\item 证明: $ A_0 $$ B $都是$ \ell_1 $的闭线性子空间, 且$ A_0+B$$\ell_1 $中稠密;
\item$ c\in\ell_1 $满足$ x_{2n-1}=0 $$ c_{2n}=2^{-n} $, 并设$ A=A_0-c $. 证明: $ c\notin A_0+B $$ A\cap B=\varnothing $, 并证明: 不存在非零的$ f\in\Star{\ell_1} $$ \alpha\in\R $使得$ A\subset\{ f\leqslant\alpha \} $$ B\subset\{ f\geqslant\alpha \} $.
\end{enumerate}
\begin{Proof}
Expand Down Expand Up @@ -476,10 +477,10 @@ \section{第8章习题}
\[
f(A_0-c)\leqslant\alpha\Longrightarrow f(A_0)\leqslant\alpha+f(c),
\]
$ f $的线性性与$ A_0 $是线性子空间可知只能$ f|_{A_0}=0 $. 同理, 由$ f(B)\geqslant\alpha $可知$ f|_B=0 $. 于是$ f\_{A_0+B}=0 $. 由$ A_0+B $$ \ell_1 $中稠密, 则$ f=0 $.\qed
$ f $的线性性与$ A_0 $是线性子空间可知只能$ f|_{A_0}=0 $. 同理, 由$ f(B)\geqslant\alpha $可知$ f|_B=0 $. 于是$ f|_{A_0+B}=0 $. 由$ A_0+B $$ \ell_1 $中稠密, 则$ f=0 $.\qed
\end{Proof}

\textbf{习题8.15(a)}\ [习题课]\ \ 本习题的目的是刻画连续线性泛函的 Hahn-Banach 延拓的唯一性问题. 设 $ E $ 是一个维数至少为 2 的赋范空间, 证明: $ E $ 的线性子空间 $ F $ 上的每个连续线性泛函有唯一的保范延拓的充分不要条件是 $ \Star{E} $ 是严格凸的.
\textbf{习题8.15(a)}\ [习题课]\ \ 本习题的目的是刻画连续线性泛函的 Hahn-Banach 延拓的唯一性问题. 设 $ E $ 是一个维数至少为 2 的赋范空间, 证明: $ E $ 的线性子空间 $ F $ 上的每个连续线性泛函有唯一的保范延拓的充分必要条件是 $ \Star{E} $ 是严格凸的.

\begin{Proof}
\textsl{充分性}. 用反证法. 若存在 $ f\in \Star{F} $ 使得它有两个不同的保范延拓 $ f_{1}, f_{2} $, 且 $ f_{1}\ne f_{2} $, 则:
Expand All @@ -490,7 +491,7 @@ \section{第8章习题}
\[
\norm{f}=\tnorm{\tilde{f}\big|_{F}}\leqslant\tnorm{\tilde{f}}\leqslant\frac{\norm{f_{1}}+\norm{f_{2}}}{2}=\norm{f},
\]
但由 $ \Star{E} $ 严格凸, 只能 $ \tnorm{\tilde{f}}\leqslant\norm{f} $. 故 $ \norm{f}<\norm{f} $.
但由 $ \Star{E} $ 严格凸, 只能 $ \tnorm{\tilde{f}}<\norm{f} $. 故 $ \norm{f}<\norm{f} $, 矛盾.

\textsl{必要性}. 用反证法. 若 $ \Star{E} $ 不是严格凸的, 即 $ \exists f\in S_{\Star{E}}\,(f=(f_{1}+f_{2})/2) $, 其中 $ \norm{f_{1}}=\norm{f_{2}}=1 $$ f_{1}\ne f_{2} $.

Expand Down Expand Up @@ -637,7 +638,7 @@ \section{第9章习题}
\]
$ \forall n\in\N $, $ f(x_{n})=0 $, 则有
\[
\norm{f}\leqslant\norm{f-f_{n}}+\norm{f_{n}}<\varepsilon + 2\abs{f_{n}(x_{n})}\leqslant \varepsilon 2\norm{f_{n}},
\norm{f}\leqslant\norm{f-f_{n}}+\norm{f_{n}}<\varepsilon + 2\abs{f_{n}(x_{n})}\leqslant \varepsilon + 2\norm{f_{n}},
\]
而又由
\[
Expand Down Expand Up @@ -757,14 +758,14 @@ \section{第9章习题}

\textbf{习题 9.16}\ [习题课]\ \ 称 Banach 空间 $ E $\textbf{一致凸}\index{Y!一致凸}的, 若对任意 $ \varepsilon>0 $, 存在 $ \delta>0 $, 使得
\[
\forall x, y\in\bar{B}_{E}\,\left(\norm{x-y}\geqslant\varepsilon\Rightarrow \norm{\frac{x+y}{2}\leqslant 1-\delta}\right).
\forall x, y\in\bar{B}_{E}\,\left(\norm{x-y}\geqslant\varepsilon\Rightarrow \norm{\frac{x+y}{2}}\leqslant 1-\delta\right).
\]
$ (x_{n})_{n\geqslant1} $ 是一致凸空间 $ E $ 中弱收敛到 $ x $ 的序列, 并有 $ \lim\limits_{n\to\infty}\norm{x_{n}}=\norm{x} $. 证明 $ (x_{n})_{n\geqslant1} $ 依范数收敛到 $ x $. 用例子说明条件 $ \lim\limits_{n\to\infty}\norm{x_{n}}=\norm{x} $ 是必需的.
\begin{Proof}
$ x_{n}\weakto x $ 有界, 因为 $ \frac{x_{n}}{\norm{x_{n}}}\weakto\frac{x}{\norm{x}} $, 不妨设 $ (x_{n})_{\geqslant1}\subset\bar{B}_{E} $.
\begin{enumerate}[(1)]
\item$ \norm{x}=0 $, 则 $ x_{n}\to x $.
\item$ \norm{x}\ne 0 $, 则 $ \exists f\in\Star{E} $, $ f(x)=\norm{x}=1 $, 其中 $ \norm{f}=1 $. 若 $ (x_{n})_{n\geqslant1} $ 是 Cauchy 列, 即 $ \norm{x_{n}, x_{m}}\to 0\ (n, m\to\infty) $, 则
\item$ \norm{x}\ne 0 $, 则 $ \exists f\in\Star{E} $, $ f(x)=\norm{x}=1 $, 其中 $ \norm{f}=1 $. 若 $ (x_{n})_{n\geqslant1} $ 是 Cauchy 列, 即 $ \norm{x_{n}- x_{m}}\to 0\ (n, m\to\infty) $, 则
\[
2\norm{x}\leftarrow f(x_{n})+f(x_{m})=f(x_{n}+x_{m})\leqslant\norm{f}\norm{x_{n}+x_{m}}\leqslant\norm{x_{m}}+x_{n}\to 2\norm{x},
\]
Expand Down Expand Up @@ -1046,7 +1047,7 @@ \section{Problem Set系列}
\[
P=\begin{bmatrix}
a & \sqrt{a(1-a)}\exp(\imag\theta) \\
\sqrt{a(1-a)}\exp(-\imag\theta) & a
\sqrt{a(1-a)}\exp(-\imag\theta) & 1-a
\end{bmatrix}, \quad a\in\R, \theta\in[0, 2\pi].
\]
进一步, 对一般的 Hilbert空间 $ H $, 有一类特殊的投影算子:
Expand Down
4 changes: 2 additions & 2 deletions chapter1.tex
Original file line number Diff line number Diff line change
Expand Up @@ -1256,7 +1256,7 @@ \section{赋范线性空间}
\end{Remark}

\begin{Proposition}[H\"older]
$ \frac{1}{p}=\sum\limits_{i=1}^n\frac{1}{p_i} $, $ f=\prod\limits_{i=1}^nf_i $时, 有
$ \frac{1}{p}=\sum\limits_{i=1}^n\frac{1}{p_i}, p_{i}\in(0, +\infty] $, $ f=\prod\limits_{i=1}^nf_i $时, 有
\[
\lVert f\rVert_p\leqslant\prod_{i=1}^n\lVert f_i\rVert_{p_i}.
\]
Expand Down Expand Up @@ -1418,7 +1418,7 @@ \section{赋范线性空间}
并赋予$ \mu\{x_i\}=1 $, 那么此时$ L_p(X) $同构于$ \ell_p $. 另一类是连续情形, 即$ X=(0,1) $, 那么$ L_p(X) $同构于$ L_p(0,1) $. 且对于任意的$ (a,b)\subset\R $, 存在等距同构$ T : L_p(0,1)\to L_p(a,b) $.

\begin{Definition}[可分性]\index{K!可分性}
若赋范空间(或度量空间)$ E $有可数稠密子集, 则称$ E $\textbf{可分的}.
若赋范空间(或度量空间) $ E $ 有可数稠密子集, 则称$ E $\textbf{可分的}.
\end{Definition}

\begin{Proposition}
Expand Down
Loading

0 comments on commit 0822922

Please sign in to comment.